LSAT and Law School Admissions Forum

Get expert LSAT preparation and law school admissions advice from PowerScore Test Preparation.

 mlhousto
  • Posts: 12
  • Joined: Oct 23, 2014
|
#17377
This question is just a general strengthen question. I chose answer option E based on my pre-phrase that the correct answer choice would strengthen the conclusion based both on money/profit (vague, I know, but i'm still working at prephrasing). E does that. The correct answer choice is C, and C was one of the first I marked off because, to me, it actually seemed to have no effect on the conclusion that:

"Some advertisers withdrew their advertisements from the publication, and this must have been because they morally disapproved of publishing salacious material."

Because it was almost as if it was a restatement of the obvious.

So unless I got the conclusion wrong, which is quite the possibility, then I don't understand why C is correct.
 David Boyle
PowerScore Staff
  • PowerScore Staff
  • Posts: 836
  • Joined: Jun 07, 2013
|
#17378
mlhousto wrote:This question is just a general strengthen question. I chose answer option E based on my pre-phrase that the correct answer choice would strengthen the conclusion based both on money/profit (vague, I know, but i'm still working at prephrasing). E does that. The correct answer choice is C, and C was one of the first I marked off because, to me, it actually seemed to have no effect on the conclusion that:

"Some advertisers withdrew their advertisements from the publication, and this must have been because they morally disapproved of publishing salacious material."

Because it was almost as if it was a restatement of the obvious.

So unless I got the conclusion wrong, which is quite the possibility, then I don't understand why C is correct.

Hello mihousto,

Answer C fits well since, if they thought their product sales would decrease if they withdrew, that means that they're doing it for the morals, not the money. (They expect to lose money from their decision to pull the ads.)
Answer E, by contrast, has, among other possible problems, the vagueness of "other income group". Poorer? Richer? What?? If answer E were structured to be more like answer C, e.g., implying that the advertisers would lose money by pulling their ads, then it might be good. But as is, answer E isn't enough like answer C.

Hope this helps,
David
 mlhousto
  • Posts: 12
  • Joined: Oct 23, 2014
|
#17390
So the strength of the language is one of the main inconsistencies of the question, since I know strengthen questions rely on definite concrete language - the stronger the language, the more validity it has?
 Andrew Ash
PowerScore Staff
  • PowerScore Staff
  • Posts: 32
  • Joined: Sep 15, 2014
|
#17417
Hi again ML,

The issue here isn't the strength of the language in the two answer choices. Instead, it's about their logical relationship to the stimulus. Let's take a step back and look at the stimulus again.

The stimulus shows us that an advertiser withdrew its advertisements after a newspaper "started concentrating on sex and violence." It then concludes that the advertisers did this because they "morally disapproved" (and that last sentence is the conclusion, you're absolutely right).

There's a big problem in this argument, though - what if the advertisers stopped advertising, not because they personally objected to the new material, but because being associated with the new material would give them bad PR? If their product was, say, DVDs of movies for children, continuing to advertise in the racy publication would probably have a disastrous effect on their sales. But their reasons for leaving would be financial, not moral - because they want to sell as many DVDs as possible. If that's true, then the conclusion is no good.

The key to a strong Strengthen prephrase is to recognize this problem in the argument. Now we're looking for an answer choice that fixes the problem. Answer choice (C) fixes the problem by showing that withdrawing is going to hurt their sales. If that's true, then they definitely didn't withdraw for financial reasons, so our conclusion, that they must have had a moral objection, looks better.

This is a classic example of strengthening a causal argument. The conclusion states that moral objections caused the withdrawal. By removing an alternate cause - namely, financial considerations - answer choice (C) strengthens the argument.

Let us know if you have any further questions on this one!

Thanks,
Andrew
 mlhousto
  • Posts: 12
  • Joined: Oct 23, 2014
|
#17424
Hi Andrew,

Your explanation completely cleared up any issues I had with the stimulus. My main issue was that I wasn't catching onto it being a causal argument. Now that you've broken it down, I can definitely see where I went wrong! C gives validity to the claim the moral claim. They withdrew even though it was at fault to them, so therefore it must have been for moral reasons.

Thanks so much!
 kcho10
  • Posts: 68
  • Joined: Nov 02, 2015
|
#32863
Hi,

The conclusion says some advertisers are motivated by moral AS WELL AS financial considerations.

C seems to suggest that maybe they are not concerned with financial considerations, which bothered me. Is it maybe implied that since they are advertisers we can conclude they have some sort of financial considerations? Is that the reason why it wouldn't weaken the conclusion?
 Adam Tyson
PowerScore Staff
  • PowerScore Staff
  • Posts: 5387
  • Joined: Apr 14, 2011
|
#32865
I think there is a good argument to be made here that you are right about the conclusion, kcho10 - while we have focused on the last statement, that the advertisers must have withdrawn due to morals, I think that statement could be viewed as supporting the claim that, therefore, advertisers are motivated by moral as well as financial factors.

Regardless, answer C helps a whole lot here, because it eliminates financial motivation as an alternate cause for their withdrawing from the newspaper under discussion. With that alternate cause eliminated, that last statement looks a lot better, and whether you view that as the main conclusion or as a premise supporting the main conclusion, strengthening it strengthens the whole thing.

What if they withdrew simply because they were going to lose money? Maybe the new content would lead to a much smaller readership, or one that was not the advertisers' target audience? That would really hurt this argument. Getting that out of the way by saying the switch will actually cost them money helps support the idea that they did it due to morals and not for money.

I think there may be some implication that advertisers are in it for money, but we don't need to bring in that outside info to have this answer help us out. The focus of the argument is on the morals, not the money, and C is the only one that helps that out.

I hope that helped!
 bk1111
  • Posts: 103
  • Joined: Apr 22, 2017
|
#43996
Hi, how can I confidently eliminate A? My thinking is that it does not do much to the argument, or if anything, it weakens the argument by suggesting the advertisers switched back to a family newspaper to maintain their financial position. It seems like I would be making assumptions to eliminate this answer choice though. Thanks!
 Jamena Pirone
PowerScore Staff
  • PowerScore Staff
  • Posts: 22
  • Joined: Feb 01, 2018
|
#44005
Hi BK,

Answer choice (A) is indeed an attractive answer choice. Since the publication in question changed from a "family newspaper" to one concentrating on "sex and violence", and in response some advertisers switched their advertisements back to other "family newspapers", it's fair to think that those advertisers must have had some sort of objection to the new sexual and violent content.

However, the conclusion of the argument is that the objection was specifically a moral one. Thus, we can only strengthen the argument with answer choices that shed light on what kind of objection the advertisers had.

You are absolutely right that answer choice (A) is wrong because it doesn't impact the argument. The argument claims to know the reason why the advertisers switched, and answer choice (A) doesn't give us any information about the advertisers' reasons.

To confidently eliminate answer choice (A), focus on the causal relationship presented in the conclusion:

Cause: Morals
Effect: Switch

Remember that to strengthen a causal argument we need the correct answer choice to do one of 5 things: Eliminate a potential alternate cause, show that the cause has occurred in other instances with the same effect, show that the effect has occurred in other instances with the same cause, demonstrate that the author has not reversed the cause and effect, or demonstrate that there is no fault with the data upon which the argument relies.

You are correct that answer choice (A) could potentially weaken the argument if the advertisers switched back to "family newspapers" for financial reasons, and you are also correct in thinking that this line of reasoning introduces uncertain assumptions. The good news is that you don't need to go that far!

Answer choice (A) simply doesn't address whether it was specifically moral objections that caused the switch. It doesn't eliminate an alternate cause, or show other instances where the cause and effect were lock-step, nor does it reaffirm the order of the causal elements or the veracity of the underlying data. If an answer choice in a causal-strengthen question doesn't do one of those things, you can feel confident that it can't be the correct answer.

Hope that helps!
 bk1111
  • Posts: 103
  • Joined: Apr 22, 2017
|
#44022
Jamena Pirone wrote:Hi BK,

Answer choice (A) is indeed an attractive answer choice. Since the publication in question changed from a "family newspaper" to one concentrating on "sex and violence", and in response some advertisers switched their advertisements back to other "family newspapers", it's fair to think that those advertisers must have had some sort of objection to the new sexual and violent content.

However, the conclusion of the argument is that the objection was specifically a moral one. Thus, we can only strengthen the argument with answer choices that shed light on what kind of objection the advertisers had.

You are absolutely right that answer choice (A) is wrong because it doesn't impact the argument. The argument claims to know the reason why the advertisers switched, and answer choice (A) doesn't give us any information about the advertisers' reasons.

To confidently eliminate answer choice (A), focus on the causal relationship presented in the conclusion:

Cause: Morals
Effect: Switch

Remember that to strengthen a causal argument we need the correct answer choice to do one of 5 things: Eliminate a potential alternate cause, show that the cause has occurred in other instances with the same effect, show that the effect has occurred in other instances with the same cause, demonstrate that the author has not reversed the cause and effect, or demonstrate that there is no fault with the data upon which the argument relies.

You are correct that answer choice (A) could potentially weaken the argument if the advertisers switched back to "family newspapers" for financial reasons, and you are also correct in thinking that this line of reasoning introduces uncertain assumptions. The good news is that you don't need to go that far!

Answer choice (A) simply doesn't address whether it was specifically moral objections that caused the switch. It doesn't eliminate an alternate cause, or show other instances where the cause and effect were lock-step, nor does it reaffirm the order of the causal elements or the veracity of the underlying data. If an answer choice in a causal-strengthen question doesn't do one of those things, you can feel confident that it can't be the correct answer.

Hope that helps!
This is very helpful, thank you!

Get the most out of your LSAT Prep Plus subscription.

Analyze and track your performance with our Testing and Analytics Package.